Những câu hỏi liên quan
Con Heo
Xem chi tiết
alibaba nguyễn
25 tháng 3 2017 lúc 9:05

Đặt: \(E=\frac{y^4}{\left(x^2+y^2\right)\left(x+y\right)}+\frac{z^4}{\left(y^2+z^2\right)\left(y+z\right)}+\frac{x^4}{\left(z^2+x^2\right)\left(z+x\right)}\)

Ta có: \(F-E=\frac{x^4-y^4}{\left(x^2+y^2\right)\left(x+y\right)}+\frac{y^4-z^4}{\left(y^2+z^2\right)\left(y+z\right)}+\frac{z^4-x^4}{\left(z^2+x^2\right)\left(z+x\right)}\)

\(=\left(x-y\right)+\left(y-z\right)+\left(z-x\right)=0\)

\(\Leftrightarrow F=E\)

Từ đó ta có:

\(2F=\frac{x^4+y^4}{\left(x^2+y^2\right)\left(x+y\right)}+\frac{y^4+z^4}{\left(y^2+z^2\right)\left(y+z\right)}+\frac{z^4+x^4}{\left(z^2+x^2\right)\left(z+x\right)}\)

\(\ge\frac{\left(x^2+y^2\right)^2}{2\left(x^2+y^2\right)\left(x+y\right)}+\frac{\left(y^2+z^2\right)^2}{2\left(y^2+z^2\right)\left(y+z\right)}+\frac{\left(z^2+x^2\right)^2}{2\left(z^2+x^2\right)\left(z+x\right)}\)

\(=\frac{\left(x^2+y^2\right)}{2\left(x+y\right)}+\frac{\left(y^2+z^2\right)}{2\left(y+z\right)}+\frac{\left(z^2+x^2\right)}{2\left(z+x\right)}\)

\(\ge\frac{\left(x+y\right)^2}{4\left(x+y\right)}+\frac{\left(y+z\right)^2}{4\left(y+z\right)}+\frac{\left(z+x\right)^2}{4\left(z+x\right)}\)

\(=\frac{x+y}{4}+\frac{y+z}{4}+\frac{z+x}{4}=\frac{1}{2}\)

\(\Rightarrow F\ge\frac{1}{4}\)

Dấu = xảy ra khi \(x=y=z=\frac{1}{3}\)

Bình luận (0)
Con Heo
25 tháng 3 2017 lúc 17:29

Bạn ơi, cho mình hỏi này

Sao có \(\frac{x^4+y^4}{\left(x^2+y^2\right)\left(x+y\right)}\ge\frac{\left(x^2+y^2\right)^2}{2\left(x^2+y^2\right)\left(x+y\right)}\)  và sao có  \(\frac{\left(x^2+y^2\right)}{2}\ge\frac{\left(x+y\right)^2}{4\left(x+y\right)}\)  

Giải đáp tận tình hộ mình nhé.

Bình luận (0)
alibaba nguyễn
25 tháng 3 2017 lúc 17:54

Cái đó là bất đẳng thức cosi đấy b

\(x^4+y^4\ge2x^2y^2\)

\(\Leftrightarrow2\left(x^4+y^4\right)\ge x^4+y^4+2x^2y^2\)

\(\Leftrightarrow2\left(x^4+y^4\right)\ge\left(x^2+y^2\right)^2\)

\(\Leftrightarrow x^4+y^4\ge\frac{\left(x^2+y^2\right)^2}{2}\) 

Bình luận (0)
Guyn
Xem chi tiết
Ayakashi
Xem chi tiết
Kiệt Nguyễn
30 tháng 7 2020 lúc 19:55

Xét: \(\frac{x^4}{\left(x^2+y^2\right)\left(x+y\right)}-\frac{y^4}{\left(x^2+y^2\right)\left(x+y\right)}=\frac{x^4-y^4}{\left(x^2+y^2\right)\left(x+y\right)}\)\(=\frac{\left(x^2+y^2\right)\left(x^2-y^2\right)}{\left(x^2+y^2\right)\left(x+y\right)}=\frac{\left(x^2+y^2\right)\left(x+y\right)\left(x-y\right)}{\left(x^2+y^2\right)\left(x+y\right)}=x-y\)(1)

Tương tự, ta có: \(\frac{y^4}{\left(y^2+z^2\right)\left(y+z\right)}-\frac{z^4}{\left(y^2+z^2\right)\left(y+z\right)}=y-z\)(2); \(\frac{z^4}{\left(z^2+x^2\right)\left(z+x\right)}-\frac{x^4}{\left(z^2+x^2\right)\left(z+x\right)}=z-x\)(3)

Cộng theo vế của 3 đẳng thức (1), (2), (3), ta được:

\(\left[\frac{x^4}{\left(x^2+y^2\right)\left(x+y\right)}+\frac{y^4}{\left(y^2+z^2\right)\left(y+z\right)}+\frac{z^4}{\left(z^2+x^2\right)\left(z+x\right)}\right]\)\(-\left[\frac{y^4}{\left(x^2+y^2\right)\left(x+y\right)}+\frac{z^4}{\left(y^2+z^2\right)\left(y+z\right)}+\frac{x^4}{\left(z^2+x^2\right)\left(z+x\right)}\right]=0\)

\(\Rightarrow\frac{x^4}{\left(x^2+y^2\right)\left(x+y\right)}+\frac{y^4}{\left(y^2+z^2\right)\left(y+z\right)}+\frac{z^4}{\left(z^2+x^2\right)\left(z+x\right)}\)\(=\frac{y^4}{\left(x^2+y^2\right)\left(x+y\right)}+\frac{z^4}{\left(y^2+z^2\right)\left(y+z\right)}+\frac{x^4}{\left(z^2+x^2\right)\left(z+x\right)}\)

Mà \(A=\frac{x^4}{\left(x^2+y^2\right)\left(x+y\right)}+\frac{y^4}{\left(y^2+z^2\right)\left(y+z\right)}+\frac{z^4}{\left(z^2+x^2\right)\left(z+x\right)}\)nên \(2A=\frac{x^4+y^4}{\left(x^2+y^2\right)\left(x+y\right)}+\frac{y^4+z^4}{\left(y^2+z^2\right)\left(y+z\right)}+\frac{z^4+x^4}{\left(z^2+x^2\right)\left(z+x\right)}\)\(\ge\frac{\frac{\left(y^2+z^2\right)^2}{2}}{\left(x^2+y^2\right)\left(x+y\right)}+\frac{\frac{\left(y^2+z^2\right)^2}{2}}{\left(y^2+z^2\right)\left(y+z\right)}+\frac{\frac{\left(z^2+x^2\right)^2}{2}}{\left(z^2+x^2\right)\left(z+x\right)}\)

\(=\frac{1}{2}\left(\frac{x^2+y^2}{x+y}+\frac{y^2+z^2}{y+z}+\frac{z^2+x^2}{z+x}\right)\)\(\ge\frac{1}{2}\left(\frac{\frac{\left(x+y\right)^2}{2}}{x+y}+\frac{\frac{\left(y+z\right)^2}{2}}{y+z}+\frac{\frac{\left(z+x\right)^2}{2}}{z+x}\right)\)\(=\frac{1}{4}\left[\left(x+y\right)+\left(y+z\right)+\left(z+x\right)\right]=\frac{1}{2}\left(x+y+z\right)=\frac{1}{2}\)(Do theo giả thiết thì x + y + z = 1)

\(\Rightarrow A\ge\frac{1}{4}\)

Đẳng thức xảy ra khi \(x=y=z=\frac{1}{3}\)

Bình luận (0)
 Khách vãng lai đã xóa
Thắng Nguyễn
12 tháng 8 2017 lúc 19:15

Bài này t làm rồi, "nhẹ" không ấy mà :|

Dự đoán khi \(x=y=z=\frac{1}{3}\Rightarrow A=\frac{1}{4}\). Ta c/m nó là GTNN của A

Áp dụng BĐT Cauchy-Schwarz dạng Engel ta có:

\(A=Σ\frac{x^4}{\left(x^2+y^2\right)\left(x+y\right)}\ge\frac{\left(x^2+y^2+z^2\right)^2}{Σ\left(x^2+y^2\right)\left(x+y\right)}\)

Cần chứng minh BĐT \(\frac{\left(x^2+y^2+z^2\right)^2}{Σ\left(x^2+y^2\right)\left(x+y\right)}\ge\frac{x+y+z}{4}\)

\(\Leftrightarrow4\left(x^2+y^2+z^2\right)^2\ge\left(x+y+z\right)Σ\left(2x^3+x^2y+x^2z\right)\)

\(\LeftrightarrowΣ\left(2x^4-3x^3y-3x^3z+6x^2y^2-2x^2yz\right)\ge0\)

\(\LeftrightarrowΣ\left(2x^4-3x^3y-3x^3z+4x^2y^2\right)+Σ\left(2x^2y^2-2x^2yz\right)\ge0\)

\(\LeftrightarrowΣ\left(x^4-3x^3y+4x^2y^2-3xy^3+y^4\right)+Σ\left(x^2z^2-2z^2xy+y^2z^2\right)\ge0\)

\(\LeftrightarrowΣ\left(x-y\right)^2\left(x^2-xy+y^2\right)+Σz^2\left(x-y\right)^2\ge0\)

BĐT cuối đúng tức ta có \(A_{Min}=\frac{1}{4}\Leftrightarrow x=y=z=\frac{1}{3}\)

P/s: Nguồn lời giải Câu hỏi của Vo Trong Duy - Toán lớp 9 - Học toán với OnlineMath, rảnh quá ngồi gõ lại :V

Bình luận (0)
Trương Thanh Nhân
30 tháng 7 2020 lúc 20:26

Cho tam giác ABC nhọn. Các đường cao AD, BE, CF cắt tại H. Qua A vẽ đường thẳng song song với BE,CF lần lượt cắt CF,BE tại P và Q. Chứng minh: PQ vuông góc với trung tuyến AM của tam giác ABC

Bình luận (0)
 Khách vãng lai đã xóa
VUX NA
Xem chi tiết
Nguyễn Việt Lâm
14 tháng 2 2022 lúc 18:17

Hướng dẫn: đặt \(A=\dfrac{y^4}{\left(x^2+y^2\right)\left(x+y\right)}+\dfrac{z^4}{\left(y^2+z^2\right)\left(y+z\right)}+\dfrac{x^4}{\left(z^2+x^2\right)\left(z+x\right)}\)

Khi đó \(F-A=x-y+y-z+z-x=0\Rightarrow F=A\)

\(\Rightarrow2F=F+A=\sum\dfrac{x^4+y^4}{\left(x^2+y^2\right)\left(x+y\right)}\ge\sum\dfrac{\left(x^2+y^2\right)^2}{2\left(x^2+y^2\right)\left(x+y\right)}\ge\sum\dfrac{\left(x+y\right)^2\left(x^2+y^2\right)}{4\left(x^2+y^2\right)\left(x+y\right)}\)

\(\Rightarrow2F\ge\dfrac{x+y+z}{2}\Rightarrow F\ge\dfrac{x+y+z}{4}\)

Bình luận (0)
Phước Nguyễn
Xem chi tiết
l҉o҉n҉g҉ d҉z҉
Xem chi tiết
Phan Nghĩa
16 tháng 7 2021 lúc 9:57

lại bị trùng rồi quỳnh ơi , https://olm.vn/hoi-dap/detail/76355556031.html

Bình luận (0)
 Khách vãng lai đã xóa
Đoàn Đức Hà
16 tháng 7 2021 lúc 9:59

Câu hỏi của Con Heo - Toán lớp 8 - Học trực tuyến OLM

Bình luận (0)
 Khách vãng lai đã xóa
๖ۣۜLuyri Vũ๖ۣۜ
Xem chi tiết
Đặng Ngọc Quỳnh
18 tháng 10 2020 lúc 12:40

Vì xyz=1\(\Rightarrow x^2\left(y+z\right)\ge2x^2\sqrt{yz}=2x\sqrt{x}\)

Tương tự \(y^2\left(z+x\right)\ge2y\sqrt{y};z^2=\left(x+y\right)\ge2z\sqrt{z}\)

\(\Rightarrow P\ge\frac{2x\sqrt{x}}{y\sqrt{y}+2z\sqrt{z}}+\frac{2y\sqrt{y}}{z\sqrt{z}+2x\sqrt{x}}+\frac{2z\sqrt{z}}{x\sqrt{x}+2y\sqrt{y}}\)

Đặt \(x\sqrt{x}+2y\sqrt{y}=a;y\sqrt{y}+2z\sqrt{z}=b;z\sqrt{z}+2x\sqrt{x}=c\)

\(\Rightarrow x\sqrt{x}=\frac{4c+a-2b}{9};y\sqrt{y}=\frac{4a+b-2c}{9};z\sqrt{z}=\frac{4b+c-2a}{9}\)

\(\Rightarrow P\ge\frac{2}{9}\left(\frac{4c+a-2b}{b}+\frac{4a+b-2c}{a}+\frac{4b+c-2a}{b}\right)\)

\(=\frac{2}{9}\text{ }\left[4\left(\frac{c}{b}+\frac{a}{c}+\frac{b}{a}\right)+\left(\frac{a}{b}+\frac{b}{c}+\frac{c}{a}\right)-6\right]\ge\frac{2}{9}\left(4.3+2-6\right)=2\)

Min P =2 khi và chỉ khi a=b=c khi va chỉ khi x=y=z=1

Bình luận (0)
 Khách vãng lai đã xóa
nguyen kim chi
Xem chi tiết
ღ๖ۣۜLinh
18 tháng 2 2020 lúc 13:38

Áp dụng bđt AM-GM ta có

\(P\ge3\sqrt[3]{\frac{xyz\left(xy+1\right)^2.\left(yz+1\right)^2.\left(zx+1\right)^2}{x^2y^2z^2\left(xy+1\right)\left(yz+1\right)\left(zx+1\right)}}=3\sqrt[3]{\frac{\left(xy+1\right)\left(yz+1\right)\left(zx+1\right)}{xyz}}=A\)

  Ta có   \(A=3\sqrt[3]{\left(\frac{xy+1}{x}\right)\left(\frac{yz+1}{y}\right)\left(\frac{zx+1}{z}\right)}=3\sqrt[3]{\left(y+\frac{1}{x}\right)\left(z+\frac{1}{y}\right)\left(x+\frac{1}{z}\right)}\)

Áp dụng bđt AM-GM ta có

\(A\ge3\sqrt[3]{8\sqrt{\frac{xyz}{xyz}}}=3.2=6\)

\(\Rightarrow P\ge6\)

Dấu "=" xảy ra khi x=y=z=\(\frac{1}{2}\)

Bình luận (0)
 Khách vãng lai đã xóa
zZz Cool Kid_new zZz
18 tháng 2 2020 lúc 18:31

Làm tiếp bài ღ๖ۣۜLinh's ๖ۣۜLinh'sღ] ★we are one★ chớ hình như bị ngược dấu ó.Do mình gà nên chỉ biết cô si mù mịt thôi ạ

\(3\sqrt[3]{\left(y+\frac{1}{x}\right)\left(z+\frac{1}{y}\right)\left(x+\frac{1}{z}\right)}\)

\(=3\sqrt[3]{\left(y+\frac{1}{4x}+\frac{1}{4x}+\frac{1}{4x}+\frac{1}{4x}\right)\left(z+\frac{1}{4y}+\frac{1}{4y}+\frac{1}{4y}+\frac{1}{4y}\right)\left(x+\frac{1}{4z}+\frac{1}{4z}+\frac{1}{4z}+\frac{1}{4z}\right)}\)

\(\ge3\sqrt[3]{5\sqrt[5]{\frac{y}{256x^4}}\cdot5\sqrt[5]{\frac{z}{256y^4}}\cdot5\sqrt[5]{\frac{x}{256z^4}}}\)

\(=3\sqrt[3]{125\sqrt[5]{\frac{xyz}{256^3\left(xyz\right)^4}}}\)

\(=15\sqrt[3]{\sqrt[5]{\frac{1}{256^3\left(xyz\right)^3}}}\)

\(\ge15\sqrt[15]{\frac{1}{256^3\cdot\left(\frac{x+y+z}{3}\right)^9}}\)

\(\ge15\sqrt[15]{\frac{1}{256^3\cdot\frac{1}{2^9}}}=\frac{15}{2}\)

Dấu "=" xảy ra tại \(x=y=z=\frac{1}{2}\)

Bình luận (0)
 Khách vãng lai đã xóa
zZz Cool Kid_new zZz
18 tháng 2 2020 lúc 20:10

Không phải ngược đâu nha mọi người,dấu bằng không xảy ra nhé!

Bình luận (0)
 Khách vãng lai đã xóa
nguyen kim chi
Xem chi tiết
Phan Nghĩa
6 tháng 8 2020 lúc 16:43

Bài này thì AM-GM thôi 

\(P=\frac{z\left(xy+1\right)^2}{y^2\left(yz+1\right)}+\frac{x\left(yz+1\right)^2}{z^2\left(zx+1\right)}+\frac{y\left(zx+1\right)^2}{x^2\left(xy+1\right)}\)

Sử dụng BĐT AM-GM cho 3 số không âm ta có :

\(\frac{z\left(xy+1\right)^2}{y^2\left(yz+1\right)^2}+\frac{x\left(yz+1\right)^2}{z^2\left(zx+1\right)}+\frac{y\left(zx+1\right)}{x^2\left(xy+1\right)}\ge3\sqrt[3]{\frac{xyz\left(xy+1\right)^2\left(yz+1\right)^2\left(zx+1\right)^2}{x^2y^2z^2\left(xy+1\right)\left(yz+1\right)\left(zx+1\right)}}\)

\(=3\sqrt[3]{\frac{\left(xy+1\right)\left(yz+1\right)\left(zx+1\right)}{xyz}}=3\sqrt[3]{\left(\frac{xy+1}{x}\right)\left(\frac{yz+1}{y}\right)\left(\frac{zx+1}{z}\right)}\)

\(=3\sqrt[3]{\left(\frac{xy}{x}+\frac{1}{x}\right)\left(\frac{yz}{y}+\frac{1}{y}\right)\left(\frac{zx}{z}+\frac{1}{z}\right)}=3\sqrt[3]{\left(y+\frac{1}{x}\right)\left(z+\frac{1}{y}\right)\left(x+\frac{1}{z}\right)}\)

Tiếp tục sử dụng AM-GM cho 2 số không âm ta được :

\(3\sqrt[3]{\left(2\sqrt[2]{y\frac{1}{x}}\right)\left(2\sqrt[2]{z\frac{1}{y}}\right)\left(2\sqrt[2]{x\frac{1}{z}}\right)}\ge3\sqrt[3]{\left(2\sqrt{\frac{y}{x}}\right)\left(2\sqrt{\frac{z}{y}}\right)\left(2\sqrt{\frac{x}{z}}\right)}\)

\(=3\sqrt[3]{8\left(\sqrt{\frac{y}{x}}.\sqrt{\frac{z}{y}}.\sqrt{\frac{x}{z}}\right)}=3\sqrt[3]{8.\sqrt{\frac{xyz}{xyz}}}=3\sqrt[3]{8}=3.2=6\)

Đẳng thức xảy ra khi và chỉ khi \(x=y=z=\frac{1}{2}\)

Vậy \(Min_P=6\)đạt được khi \(x=y=z=\frac{1}{2}\)

Bình luận (0)
 Khách vãng lai đã xóa